Solving the Legendary IMO Problem 6 in 8 minutes | International Mathematical Olympiad 1988

Sdílet
Vložit
  • čas přidán 14. 01. 2021
  • #IMO #IMO1988 #MathOlympiad
    Here is the solution to the Legendary Problem 6 of IMO 1988!!
    ------------------------------------------------
    Follow my Instagram page for more Maths :)
    Link: / lets.think.critically
    ------------------------------------------------
    I share Maths problems and Maths topics from well-known contests, exams and also from viewers around the world. Apart from sharing solutions to these problems, I also share my intuitions and first thoughts when I tried to solve these problems.
    Subscribe: czcams.com/users/letsthinkcr...
    Email me (address in video) your suggestions! lets.think.critically.27@gmail.com

Komentáře • 365

  • @GothicKin
    @GothicKin Před 3 lety +331

    The critical part is A1 is not 0 because B^2 - k can't vanish. Blink and you miss it, still a great job laying out the proof!

    • @MarioRossi-sh4uk
      @MarioRossi-sh4uk Před 3 lety +17

      Yes, sure. Well said.
      If you omit that detail, you may start the demonstration by stating °let's assume k is a perfect square°, and then conclude that by contradiction k must not be a perfect square.

    • @Guillaume_Paczek
      @Guillaume_Paczek Před 3 lety +1

      ah yes thanks, i was wondering the link with the fact we forbid k to be a square

    • @GothicKin
      @GothicKin Před 3 lety

      @@MarioRossi-sh4uk Probably both elegant, I wonder if that would complicate the proof or make it simpler

    • @GauravPandeyIISc
      @GauravPandeyIISc Před 3 lety +12

      This proves a stronger claim: Not only k is a square, it is the square of the smaller of A and B. Effectively, this proves that (a^2 + b^2)/(ab + 1) = b^2 if a>=b and a,b are both positive integers. Solving the above, we get that (b^3, b) are the only solutions to the above equation.

    • @werdo9638
      @werdo9638 Před 2 lety +7

      @@GauravPandeyIISc thats actually not true. (a,b)=(30,8) would be a counterexample

  • @buxeessingh2571
    @buxeessingh2571 Před 3 lety +416

    Remember: even Terry Tao did not find a complete proof to this question.

    • @cr1216
      @cr1216 Před 3 lety +113

      In the limited time of the exam though. Remember in that amount of time he had to do two other questions as well, which he did well.

    • @timothy6955
      @timothy6955 Před 3 lety +134

      and he was 12 or 13 years old

    • @kevinm1317
      @kevinm1317 Před 3 lety +75

      Also remember that by today's standards, this is a relatively easy problem compared to then. Its about as standard as Vieta Jumping gets, but Vieta Jumping was nearly unheard of back then, which is why this problem is so famous

    • @zawadulhoque4511
      @zawadulhoque4511 Před 3 lety +9

      @@kevinm1317 yeah today it would make a hard p1/easy p2

    • @vaibhavsingh1113
      @vaibhavsingh1113 Před 3 lety +20

      Terrance Tao won Bronze medal in IMO at age of 11
      and I failed to even qualify for National team at age of 15

  • @PranavGarg_
    @PranavGarg_ Před 3 lety +8

    This was the second video I watched from this channel and it was a good understandable solution. Just subscribed.

  • @ignaciobenjamingarridoboba2071

    This solve is brilliant, always you assume there is a minimum (a,b) when the equation is not a perfect square, you'll always find out a smaller number than the minimum

  • @kephalopod3054
    @kephalopod3054 Před 3 lety +29

    I saw somewhere that this problem is a particular case of the nice generalization (much harder to prove):
    Let a, b positive integers. Prove that
    if (ab)^(n-1) + 1 | a^n + b^n, then
    (a^n + b^n) / ((ab)^(n-1) + 1) is a perfect n-th power.

    • @dimitrismelas508
      @dimitrismelas508 Před 3 lety +1

      This looks interesting, where did you find it?

    • @mattiascardecchia799
      @mattiascardecchia799 Před 3 lety +39

      I have a wonderful proof of this, but I'm afraid it doesn't fit within the margin of this CZcams comment section...

    • @keescanalfp5143
      @keescanalfp5143 Před 2 lety +1

      @@mattiascardecchia799, quite interesting. could you give an indication for the starting direction of that proof.

    • @themathsgeek8528
      @themathsgeek8528 Před 2 lety +1

      @@mattiascardecchia799 lol

    • @imauz1127
      @imauz1127 Před 3 měsíci +2

      @keescanalfp5143 it’s a fermat reference

  • @msk4246
    @msk4246 Před 3 lety +33

    Elegance at its peak...... 🙏🙏🙏🙏🙏

  • @ratulee
    @ratulee Před 6 měsíci +7

    It can be solved with elementary number theory without Vieta jumping. If you modify the rest of the theorem a little bit.
    The size of b is between ak-a and ak.
    Set b = ak - r (0

  • @pbj4184
    @pbj4184 Před 3 lety +193

    5:47 I would like to add a few more steps here so that the jump may become clearer for those who didn't get it. A1B+1>0
    => A1B > -1
    As A1 was proved to be an integer and we know B is an integer as well (natural to be more specific), A1*B must also be integral. Hence A1B can be 0 at minimum as that is the next integer after 0
    => A1B >= 0
    A1 was shown not to be 0 and B is natural so it can't be 0. Hence since both A1 and B are non-zero, their product must also be non-zero and therefore it can only be >0
    => A1*B > 0
    Now since B is positive by virtue of being natural, A1 must also be positive. QED

    • @nicholasroberts2933
      @nicholasroberts2933 Před 3 lety +7

      Other viewers will appreciate this comment. Thank you

    • @lewischeung868
      @lewischeung868 Před 3 lety +7

      This comment clearly saves the proof :)

    • @tioa.p.1058
      @tioa.p.1058 Před 3 lety +1

      thanks

    • @pedrojose392
      @pedrojose392 Před 3 lety +6

      @@lewischeung868 , I do not agree, that it saves the proof. The proof itselve is clean and safety. It was just a step so easy, that he did not waste time explaining. The proof it is so pretty, excellent. Nothing to repair.

    • @lewischeung868
      @lewischeung868 Před 3 lety +3

      @@pedrojose392 i am sorry to tell you that i don't agree with your point. The logic behind proof by infinite descent is to show "we can generate a smaller counter example from a given counter example". However, natural number has a lower bound, which is "1" accordingly. We can't accept an infinite descent algorithm for such problem. Then, a contradiction arises.
      Why this step is necessary? If we can't make sure A1 is positive, we cannot say (A1,B) is a possible candidate for a smaller counter example. Our infinite descent algorithm cannot bee carried out and eventually the proof is meaningless.
      I hope my terrible english can persuade you the reason behind. :)

  • @littlefermat
    @littlefermat Před 3 lety +26

    The problem that you'll see in every NT book for math Olympiad.

  • @kayson971
    @kayson971 Před 3 lety +227

    I remember seeing this problem in one of my math sessions disguised as a harmless question
    And the whole class was struggling to solve it

    • @4ltrz555
      @4ltrz555 Před 3 lety +68

      Does your math teacher hate u guys lmao

    • @kayson971
      @kayson971 Před 3 lety +61

      @@4ltrz555 I believe it was the coordinator that gave the question, so the funnier thing is that the teacher didn't know that this was an imo question either

    • @4ltrz555
      @4ltrz555 Před 3 lety +6

      @@kayson971 haha

    • @yatharthsingh5349
      @yatharthsingh5349 Před 3 lety +4

      Same, lmao.

    • @yat_ii
      @yat_ii Před 2 lety +11

      we do a little trolling

  • @alexandergolys2087
    @alexandergolys2087 Před 3 lety +4

    Such a cool proof, thanks!

  • @Luarhackererreape
    @Luarhackererreape Před 3 lety +12

    Excelente bro! me gusta que pongas los subtítulos en español! Ganaste un suscriptor :)

  • @pahularora9642
    @pahularora9642 Před 3 lety +7

    Amazing solution....Loved it..

  • @mohamedazizghorbel6413
    @mohamedazizghorbel6413 Před 2 lety +3

    Thank you for such a nice work , all my Support ❤️

  • @ren200758
    @ren200758 Před 9 měsíci +7

    struggled with the contradiction a bit in the end. the trick is in order for this not to contradict, B^2 must equal k. nice trick!

  • @aua6330
    @aua6330 Před rokem +1

    Perfectly done, thank you.

  • @danielontheedge
    @danielontheedge Před 3 lety +155

    It's quite hard to find examples of a and b that satisfy this... One example is (1, 1)

    • @lucacastenetto1230
      @lucacastenetto1230 Před 3 lety +45

      If you take a=b³ those are all the soluzions i think

    • @patrickng8974
      @patrickng8974 Před 3 lety +13

      There are endless number of examples: 1,1;2,8;3,27;4,64;5,125......

    • @Qermaq
      @Qermaq Před 3 lety +4

      Look at the thread I started a few weeks ago. People have posted a lot of insights.

    • @patrickng8974
      @patrickng8974 Před 3 lety +10

      other than setting a = b^3, another set of solutions can be found by setting a = n^2 and k = n^3 where k(ab+1) = a^2+b^2

    • @grammairiennase624
      @grammairiennase624 Před 3 lety +1

      @Luca Castenetto
      Wrong, (0, k) or (k, 0) for all k != 0 is good, too.
      Infinity of trivial examples, not following the a = b^3 or b = a^3 rule.

  • @user-ip4bm4xp2q
    @user-ip4bm4xp2q Před 3 lety +33

    Thanks,author. Please make more content like that(I'm the Russian olympiad participant)

  • @ary480
    @ary480 Před 3 lety +1

    This channel will get 1 million by December 2021

  • @webtoon1121
    @webtoon1121 Před 3 lety +19

    Thank you very much. this idea is very helpful for my problem that ask me to prove for positve x,y if (x²+y²+6)/xy integer then it must be perfect cubes

  • @MrCarlosmario22
    @MrCarlosmario22 Před rokem +1

    Exelente razonamiento. Muchas Gracias.

  • @theevilmathematician
    @theevilmathematician Před 3 lety +7

    Very interesting number theory problem.

  • @mahxylim7983
    @mahxylim7983 Před 9 měsíci

    Nice explaination!

  • @HaotianWu-bm2fx
    @HaotianWu-bm2fx Před 3 měsíci

    A very clear explanation👍

  • @jasonleelawlight
    @jasonleelawlight Před 5 měsíci

    This method is very interesting! I didn’t expect this setup could lead to a solution. I need to watch again to better evaluate what role “being a perfect square” plays in solving this problem.

    • @jasonleelawlight
      @jasonleelawlight Před 5 měsíci

      I think I figured it out, basically if we twist the problem a bit and assume we are only given that “k is a natural number” and nothing is said about perfect square, we can still find that the solution is actually limited to a specific structure, i.e. k must be B^2 and A must be B^3, as this is the only way this whole thing can hold up.

    • @unemployed5373
      @unemployed5373 Před 3 měsíci +1

      Thank you for clearing things up, I had no idea how this solution explains k being a perfect square.

  • @KJ-zs7pi
    @KJ-zs7pi Před 3 lety +3

    Awesome😃

  • @biscuitnerd7243
    @biscuitnerd7243 Před 3 měsíci +2

    So yay we are done :D

  • @sparkaks-gr8647
    @sparkaks-gr8647 Před 3 lety +1

    Woah its great

  • @allaboutcommands4984
    @allaboutcommands4984 Před 3 lety

    you make it look so easy lol ^^'

  • @trungnhanpham7694
    @trungnhanpham7694 Před 2 měsíci +1

    Me, a 14 years old, suck at math, watching this, having no idea what he's talking about, but its very interesting

  • @SuperYoonHo
    @SuperYoonHo Před rokem +2

    Thank you!!!!!!!!!!!!!!!!!!!!

  • @prithujsarkar2010
    @prithujsarkar2010 Před 3 lety +11

    That's soooo cool

  • @abhinavpj1729
    @abhinavpj1729 Před 3 lety +1

    Awesome🥰

  • @quantumgaming9180
    @quantumgaming9180 Před 5 měsíci +2

    Dude, my college professor posted on his facebook page your video here. Glad I was able to find a simpler proof of this problem

  • @NakSrea84
    @NakSrea84 Před 3 lety +1

    Wow so good teacher I will teach my students the same to you
    Because your skill is very nice

  • @padraiggluck2980
    @padraiggluck2980 Před 2 lety

    Very nice. 👍

  • @pauselab5569
    @pauselab5569 Před 4 měsíci

    A nice trick is to quickly abuse symmetry and transform this into symmetric polynomials form. Then it becomes a lot easier but still hard to solve without the hell a lot of ring theory

  • @brendanchamberlain9388
    @brendanchamberlain9388 Před 3 lety +5

    really good

  • @user-lr8od4uz1n
    @user-lr8od4uz1n Před 9 měsíci

    Beautiful

  • @muhendisgenc8216
    @muhendisgenc8216 Před 2 lety

    Wow nice one

  • @ranjitprasad2155
    @ranjitprasad2155 Před 3 lety +2

    Those 11 students , 🤯🤯

  • @iainfulton3781
    @iainfulton3781 Před rokem +16

    There's only one negative integer solution to the equation which is -5. The 8 non reducible sets of a and b are (-1,2) (-1,3) (2,-1) (3,-1) (1,-2) (1,-3) (-2,1) and (-3,1) and with these you can Vieta jump to larger absolute values. Like -5(3) - (-1) yields -14,3

  • @anshugupta793
    @anshugupta793 Před rokem

    Awesome

  • @subramanyakarthik5843
    @subramanyakarthik5843 Před 3 měsíci

    This equation satisfies only if A and B are perfect squares when substituting to that equation will result to a perfect solution😊

  • @Uknowwhois
    @Uknowwhois Před rokem

    Bro has proved hardest imo problem by contradiction

  • @mathisnotforthefaintofheart

    Sometimes I think it is even harder to come up with a theorem like this...

  • @TanvirSami-jo4tx
    @TanvirSami-jo4tx Před 2 měsíci

    I did it(vieta jumping),Andromida and milkiway,cassiopeia

  • @omaralvarezzaleta4728
    @omaralvarezzaleta4728 Před 3 lety

    Soy asesor de olimpiadas de matemáticas en prepa, nivel regional,Chiapas México

  • @mkj1887
    @mkj1887 Před 2 lety +2

    My preferred wording of the problem is that the given expression is not a prime. Then: case 1 is that the expression is not an integer, in which case it certainly is not a prime. case 2 is where we show that it must be a square, and a square is never a prime. QED.

  • @iainfulton3781
    @iainfulton3781 Před rokem +3

    The pairs of integers that fit the equation are x^(2n-1) - (n-2)x^(2n-5) + T(n-4)x^(2n-9) - TT(n-6)x^(2n-13) + TTT(n-8)x^(2n-17) - TTTT(n-10)x^(2n-21) + ... where T(n) is the triangle number TT(n) is the triangle number of the triangle numbers and TTT(n) is the triangle number of the triangle numbers of the triangle numbers and so on. If you substitute n = n - 1 you get the other pair and if the power becomes negative you stop the formula. So if n = 11 you get a=(x^21 - 9x^17 + 28x^13 - 35x^9+15x^5- x) b= (x^19 - 8x^15 + 21x^11 - 20x^7 + 5x^3) cause T(11-4)=28 TT(11-6) = 1+3+6+10+15 =35 TTT(11-8) = 1+1+3+1+3+6=15 TTTT(11-10) =1 and T(10-4)=21 TT(10-6)=1+3+6+10=20 TTT(10-8) = 1+1+3=5. All the coefficients add to either (1,1) (1,0) (0,1) (0,-1) (-1,0) or (-1,-1) so that x = 1 will result in 1.

    • @spiderjerusalem4009
      @spiderjerusalem4009 Před 7 měsíci

      from where did u get all these?

    • @victory6468
      @victory6468 Před 5 měsíci +1

      @@spiderjerusalem4009 proof by intimidation, write a whole bunch of mathematical jargon no one can read, and no one will doubt your proof

    • @spiderjerusalem4009
      @spiderjerusalem4009 Před 5 měsíci

      @@victory6468 the jargons are comprehensible. It's just the derivations, where it came from were utter vague

  • @gauranshbansal
    @gauranshbansal Před 10 měsíci

    I really like your accent, that stereotypical Asian accent (I mean it in a good way, I'm not being racist, I'm Asian too) makes me much more comfortable dunno, if I'm the only one

  • @Alberto-nz6er
    @Alberto-nz6er Před 3 měsíci

    One of the students who solved the problem, is now the mayor of Bucharest, the city I’m living in

  • @MegaRainnyday
    @MegaRainnyday Před 3 lety +3

    From your proof, we can strengthen the statement by replacing a perfect square with b^2, right?
    Edit: It also need to add an assumption b

    • @zerosumgame9071
      @zerosumgame9071 Před 3 lety +2

      No it’s not b^2. For example (8,30) is a solution which equals 4, which is not the square of either input

    • @BrunoVisnadi1
      @BrunoVisnadi1 Před 3 lety

      We only know that for sure if a+b is mimimal

  • @babulalyogi1952
    @babulalyogi1952 Před 2 lety +1

    Well I solved it in few minutes and astonishingly my solution was also correct...
    Can I send it to someone to verify it????

  • @andreadevescovi4166
    @andreadevescovi4166 Před rokem

    If ab+1 divides a^2+b^2 then b^2=a/b (it is simple: divide (a^2+b^2) by ab+1 and to make the rest=0 it is necessary b^2=a/b)
    Then b^2=a/b --> b^3=a. ----> substituting in (a^2+b^2)/(ab+1) --> (a^2+a^6)/(a^4+1)=(a^2(a^4+1))/(a^4+1)=a^2.

    • @notmymain2256
      @notmymain2256 Před 9 měsíci

      Long division "divisibility" works on polynomials, you're confusing divisibility on every a, b and divisibility on specific a, b

    • @notmymain2256
      @notmymain2256 Před 9 měsíci

      Also, don't you think it's a problem if you get a result like that since, by symmetry, you could conclude b=a^3 and so a=b=1 only solution? (btw you can easily see (2, 8) is another solution)

    • @antonioorlando5246
      @antonioorlando5246 Před 9 měsíci

      I am not able to find the condition b^2=a/b. By dividing a^b+b^2 by ab+1 the rest is a^2-a^2b-a+b^2. Now how do you elaborate on a^2-a^2b-a+b^2=0 to get that there must be b^2=a/b. Thanks

    • @BossDropbear
      @BossDropbear Před 5 měsíci +1

      Just to express differently ... (a^2+b^2)/(ab+1)=k, where a,b,k all pos integers.
      So need k*(ab+1)=kab+k to equal a^2+b^2.
      Hence need (1) kab=a^2 i.e. kb=a and (2) k=b^2.
      Substituting in for k in eq1, then kb=(b^2)*b=b^3=a.
      With a=b^3 we substitute and simplify:
      a^2+b^2 =b^6+b^2 =(b^2)*(b^4+1)
      ab+1 = b^4+1
      So ratio = b^2 = k.
      Done.

  • @represent409
    @represent409 Před 3 lety +2

    nice

  • @omaralvarezzaleta4728
    @omaralvarezzaleta4728 Před 3 lety

    Muy bueno

  • @garydetlefs6095
    @garydetlefs6095 Před 4 měsíci

    I enjoy your videos but I am very curious about seeing things in the flesh so I was curious as to what numbers actually satisfy this condition. It took me about 10 seconds to write a line of maple code to produce the results and it is interesting to see that any two numbers x and x cubed will satisfy the conditions for a and b
    The only pairs less than a thousand which also satisfy this condition are
    (30,8)...(112,30)...(240,27)...(418,112)

  • @jmart474
    @jmart474 Před 3 lety +3

    I found an easy solution, but of course there must be something wrong with my assumption.
    a^2+b^2 = k (ab+1)
    a^2+b^2 = kab + k
    Then I consider !!!
    a^2 = kab
    b^2 = k
    So k=a/b and k=b^2 and thus a = b^3
    Substituting (b^6+b^2)/(b^4+1) = b^2
    Which is a perfect square
    Hope that you can comment on this solution.

    • @sinistergaming1418
      @sinistergaming1418 Před 2 lety +3

      How is that possible as k cannot be equated to b^2 as we didnt prove k is a perfect square ,the main motive is to prove k is a perfect square so we cannot assume it

    • @aaykat6078
      @aaykat6078 Před 2 lety

      @@sinistergaming1418 it doesn't really assume that k is a perfect square
      a+b
      ------ = n
      c+d
      if a/c =n
      Then b/d also equal n
      9+18
      -------- = 3
      3+6
      9/3=3,18/6=3
      27/9=3
      This is how division and ratio works, since we have unknowns, it's safe to say a²/ab = k, and same with b²/1= k
      Although there will be times where the solution isn't like this, so i guess this is just possible answers

  • @Qermaq
    @Qermaq Před 3 lety +1

    What i find interesting are the answers I find: a and/or b = 0, or a = b^3, or a^3 = b, and that seems to be it.

    • @petersievert6830
      @petersievert6830 Před 3 lety +1

      Would be sweet to proof this thing by showing, that these are the only solutions possible, because then it easily breaks down to k=b^2 (respectively k=a^2)
      There might be a way to show this in a way, that any prime factor that is in a must also be in b and vice versa and once you are there, then conclude that the exponent must be exactly 3.

    • @vindex7
      @vindex7 Před 3 lety +3

      Unfortunately only some of the solutions are of this form. Take for example a=30, b=8.

    • @petersievert6830
      @petersievert6830 Před 3 lety

      @@vindex7 thanks for pointing this out.

    • @Qermaq
      @Qermaq Před 3 lety

      @@vindex7 Yep, I'm finding 30, 112 and 27, 240 as well. As 8 and 27 are both cubes I suspect 8, 30 and 27, 240 are related. But 30,112 is a mystery.

    • @bsmith6276
      @bsmith6276 Před 3 lety +7

      I found this: Let (a, b) be any solution pair with a>b and let s = (a^2+b^2)/(ab+1). Then another solution can be derived by creating solution pair (s*a-b, a). So if we start with a trivial (a,0) solution then that generates (a^3, a). Then from (a^3,a) we can generate (a^5-a, a^3) as another solution. And of course we can keep going to generate larger solutions.

  • @peponi3456
    @peponi3456 Před 3 lety +3

    7:29
    Why does this cobtradiction arises because of k not being perfectly squared?
    If k was a perfect square then it would be
    A1> or =0 so A1

    • @anshumanagrawal346
      @anshumanagrawal346 Před 3 lety

      Then B^2 - k =0

    • @peponi3456
      @peponi3456 Před 3 lety +3

      @@anshumanagrawal346 if k not being a perfect square leads to a contradiction then k being a perfect square must not lead to a contradiction. The contradiction is a2

    • @ericzhu6620
      @ericzhu6620 Před 2 lety +1

      @@peponi3456 4:50 from this term we can see if B^2-k = 0 then A1=0, which is not a natural number, which does not lead to a contradiction at the end since A1 is never valid as a solution, in the actual solution A1 leads to contradiction because A1 > 0, which contradicts to the assumption "(A+B) is minimal"

    • @123integration9
      @123integration9 Před rokem

      Yaa man you can assume k as not a trangular number and with the contradiction you can prove that k is a trangula number.

  • @AmazingVideoGaming
    @AmazingVideoGaming Před 3 lety +2

    *And I thought my handwriting was bad!*

  • @navjotsayal
    @navjotsayal Před 3 lety

    ab+1|a²+b² or (ab+1)|(a²+b²)

  • @happyrogue7146
    @happyrogue7146 Před 3 lety

    this is the shortest video on this problem

  • @LongNguyen-lg4zi
    @LongNguyen-lg4zi Před 3 lety +4

    why can you conclude k is a perfect square? you just proved that for every k there is only one satisfying set

    • @bwobkjobrien2508
      @bwobkjobrien2508 Před 2 lety

      The case of the repeated root would require A^2 = B^2 - k since it would be when A1 = A (the same equation used in the proof in the video), but k = B^2 - A^2 is only positive when b>a, which is false by assumption.

  • @Miguel-xd7xp
    @Miguel-xd7xp Před 3 lety +4

    Vieta jumping is the elegant solution, but the others guys who solved this problem with which solution did it? 🤔

    • @prithujsarkar2010
      @prithujsarkar2010 Před 3 lety

      most probably all of the people who got a 7 did vieta jump

    • @wayneyam1262
      @wayneyam1262 Před 3 lety +1

      @@prithujsarkar2010 nah, numberphile said only one solved that problem perfectly

    • @Miguel-xd7xp
      @Miguel-xd7xp Před 3 lety +2

      @@wayneyam1262 I don't think so, if you search in the IMO web site, there were people who got 42 but only one guy got a special prize :p

    • @pbj4184
      @pbj4184 Před 3 lety

      @@Miguel-xd7xp And that guy did it this way :)

  • @iainfulton3781
    @iainfulton3781 Před rokem

    Turn on postifications

  • @mathscornersomilpitliya4243

    It's better version is to prove it =(gcd(a,b))^2

  • @lewischeung868
    @lewischeung868 Před 3 lety +3

    May I ask how to make sure A1 is a positive number?

    • @pbj4184
      @pbj4184 Před 3 lety +2

      I posted a comment about this. Hope it helps

  • @ankitkumar-pw6pu
    @ankitkumar-pw6pu Před 3 lety

    Sir I don't understand any thing what should I do to understand this solution I mean any basic available

  • @migry
    @migry Před 3 měsíci

    I thought that the vertical line was the C computer language “or” operator 😅

  • @marlongrau246
    @marlongrau246 Před rokem

    I'm sorry is this related to phytarean triples? It doesn't seemed to be.

  • @Dinosaur-xj3kx
    @Dinosaur-xj3kx Před 6 měsíci

    What does ab+1 | a²+b² mean ? Why we are using vertical line between two equations?

  • @suuujuuus
    @suuujuuus Před 8 měsíci

    How do we know that A, the root, is an Integer, i.e. a non floating point number in proof that A1 is in Z?
    Also, that A1 is >0 comes from A1B+1>0 A1>(-1)/B which gets us A1>(-1) since B is an Integer. Since we just showed that A1 is a whole number and we assumed for our proof by contradiction that A1 /= 0, otherwise k would be an Integer square, A1 has to be in IN/0. Therefore A1>0.
    Feel like you not only skipped a lot of steps there, but also presented them in a wrong order.

  • @aaryan8104
    @aaryan8104 Před 5 měsíci +1

    i dont know a lot on how to solve these type of questions or how these even work rather but heres how i solved,please just tell me if im wrong anywhere(i certainly will be)
    let us assume
    a^2+b^2/ab+1=p where p is a natural number is not a square ----(1)
    ab+1/a^2+b^2=y which is a natural number
    ab+1=(a^2+b^2)(y)
    (a^2+b^2)(y)/(a^2+b^2)=p
    1/y=p
    y=1/p
    but according to (1) p is a natural number but i/natural number is not a natural number
    therefore our assumption is false and p is a square number

  • @marlongrau246
    @marlongrau246 Před rokem

    Okay, there must be some values of a and b when divides by ab+1 gives you a 0 remainder. Okay. Please provide some samples.

  • @mustydustard
    @mustydustard Před 3 měsíci

    how do you know its an integer

  • @chanderkumar7061
    @chanderkumar7061 Před 3 měsíci

    Sir my answer firstly distributed ab +1 in a^2+b^2 take a>=b so a^2 greater than ab +1 so if we divide than remainder will be -a/b and if we divide b^2/ab+1 remainder will be b^2 net remainder will be zero -a/b+b^2=0 so a=b^3 if we put this value in expression we got b^2 which is perfect square ... Thank you I am from india

  • @sageofsixpack226
    @sageofsixpack226 Před 3 lety +9

    5:41 what if B was negative? Than if A1 is negative we're going to end up having positive denominator and, thus, k is positive as well

    • @earthlington2
      @earthlington2 Před 3 lety +35

      A and B are both natural numbers, so B can't be negative

    • @anshumanagrawal346
      @anshumanagrawal346 Před 3 lety

      The question states that a and b are strictly positive integers

  • @mathsinmo4372
    @mathsinmo4372 Před 6 měsíci +2

    hey please check this solution a²+b² can be written as (a²+b²)(1+ab) - ab(a²+b²) and as (1+ab)|(a²+b²) then ab(a²+b²) should be equal to zero In case 1, when a² + b² = 0, the expression (a² + b²)/(1 + ab) simplifies to 0/(1 + ab) = 0, which is indeed a perfect square.
    In case 2, when ab = 0, the expression (a² + b²)/(1 + ab) simplifies to (a² + b²)/(1 + 0) = (a² + b²)/1 = a² + b². Since ab = 0, it follows that a² + b² = (a + b)², which is a perfect square.
    Therefore, based on these two cases, it can be concluded that for any values of a and b, the expression (a² + b²)/(1 + ab) is always a perfect square.

    • @francescogennaro5873
      @francescogennaro5873 Před 6 měsíci +1

      you just showed that it works if a = 0 or b = 0, not for any case

    • @ostdog9385
      @ostdog9385 Před 6 měsíci +1

      Your first step is wrong. You can only say ab(a^2+b^2) is divisible by ab+1, not that it is zero. For example 2|8, but 8=(8)(2)-1(8), but 8isnt 0.

    • @mathsinmo4372
      @mathsinmo4372 Před 6 měsíci

      @@ostdog9385 i am already wrong just fun see the divisor must be greater then the remainder that is 1 + ab > -ab(a²+b²)

  • @bilkishchowdhury8318
    @bilkishchowdhury8318 Před rokem

    4:36 How are A,B (the minimum roots of the equation) known to be integers?

    • @jilow
      @jilow Před 10 měsíci

      It's not like that.
      The problem is claiming that ALL natural solutions also happen to produce a perfect square.
      So the guy says let's say we find a solution that meets all the criteria a,b are naturals and that those two expressions divide. Suppose we find a solution and not just any solution we find the smallest solution. Which of course there will be.
      Assume we have the smallest solution that is NOT a perfect square then this proofs shows if that were the case you could always make a smaller one..which is a contradiction. Therefore, it must be a perfect square.

  • @fernandoalmer3312
    @fernandoalmer3312 Před rokem +1

    Can anyone explain what is the relation between the assumption that k is not a perfect square and the minimality of the roots?

    • @florentinmunch6769
      @florentinmunch6769 Před 11 měsíci +3

      k not square was used to deduce that A1 is not zero, and hence positive by a later argument. Minimality of roots is a fancy formulation of induction. Having (A,B) a solution, it is shown that (A1,B) is a smaller solution which is a contradiction assuming that (A,B) is a minimal solution. Here, positivity of A1 is needed so that (A1,B) is a proper solution. In other words, the Vieta jumping produces smaller and smaller roots, hitting zero at some point. But hitting zero is only possible if k is square.

    • @spiderjerusalem4009
      @spiderjerusalem4009 Před 7 měsíci

      the root A_1 = (B²-k)/A. k not being square means that can't vanish

  • @icicles_
    @icicles_ Před rokem

    It's not that bad considering current USAMO level

  • @azamatbagatov4933
    @azamatbagatov4933 Před 8 měsíci

    Elegant proof but disturbing at the same time lol

  • @daemonturk
    @daemonturk Před rokem

    Why does the proof by contradiction imply that the assumption about k not being a perfect square is false? It could also imply the assumption about k being a natural number is false. Why is the proof sound?

  • @sawyersmith5373
    @sawyersmith5373 Před 2 lety

    Why can't the contradiction arise for perfect square k?

  • @adithya3642
    @adithya3642 Před 4 měsíci

    6:01 im confused, what if A1 and B are both negative? also how does it being positive tell us its an integer?
    not sure how you got A1+B > 0

    • @benkahtan6802
      @benkahtan6802 Před měsícem

      Since A1 = kB - A, where k, B, and A are all integers, we know A1 is an integer.
      We know A1 = (B^2 - k) / A by Vieta's formulas. Since B is an integer, and we are supposing k is not a square, then B^2 - k ≠ 0, so A1 ≠ 0.
      Combining the above two results, we know that A1 is a non-zero integer.
      We know (A1^2 + B^2) / (A1 * B + 1) = k > 0. Since the numerator A1^2 + B^2 > 0, then this quotient is only positive if the denominator A1 * B + 1 is also positive.
      A1 * B + 1 > 0 implies A1 * B > -1. We know B > 0 since it was defined that way when setting up the problem. We know from above that A1 ≠ 0. Since A1 and B are integers, their product can't be between -1 and 0. So A1 * B can't be less than 0 (-1, -2, -3, ...) and it can't be 0, so it must be greater than 0.

  • @10names55
    @10names55 Před 2 lety

    Why you got that A >= B

  • @ricardocesardasilvagomes9549

    🤩🤩🤩🤩🤩🤩🤩🤩🤩👏👏👏

  • @PracticeMakePerfectMuslim93
    @PracticeMakePerfectMuslim93 Před 10 měsíci

    why did not predict the that a perfect square is positive number like 0 greater rather than just tell it a perfect squareroot

  • @dr.merlot1532
    @dr.merlot1532 Před 3 lety

    where are these 11 kids who solved this?

  • @vasantkumarmishra3537
    @vasantkumarmishra3537 Před 3 lety +4

    Didn't get How U have done for perfect Square u shows it is positive

    • @RogerSmith-ee4zi
      @RogerSmith-ee4zi Před 3 lety

      A perfect square is always positive.
      There is no perfect square that is negative
      Because negative numbers don't have roots
      + Into + = +
      - into - = +

    • @benyseus6325
      @benyseus6325 Před 3 lety +2

      @@RogerSmith-ee4zi engineers would beg to differ. Take a trip to the complex plane yo, 😎😎😎

    • @RogerSmith-ee4zi
      @RogerSmith-ee4zi Před 3 lety

      @@benyseus6325 yes in the complex plane we have the imaginary number calculations

  • @anthonypua8039
    @anthonypua8039 Před 3 lety +8

    The proof was based on the case where a + b is the minimal being assumed. What about the rest of the cases where a + b is not the minimal ?

    • @kenthchen
      @kenthchen Před 3 lety +8

      This uses a proof by contradiction, where you assume that (a²+b²)/(ab+1) is not a perfect square, then you draw a contradiction from the initial assumption. In this case, we proved that there is no minimal solution that equals a non-square. Since a and b are in the natural numbers, this effectively proves that there are no solutions at all which equal a non-square. This doesn't only prove a minimal case, it proves that all the cases must lead to a perfect square.

    • @pbj4184
      @pbj4184 Před 3 lety +11

      We didn't _assume_ a+b was minimal. We chose se (A,B) such that it gave minimal A+B as we know _a_ pair giving minimal sum must exist because there is always a minimum value in a list of integers (the list of A+B here) Then we showed that there exists a pair that gives an even smaller sum which is impossible since we chose the pair which already
      had the lowest possible sum. Therefore a logical contradiction happened and so some assumption must have been wrong. There was only one that k wasn't a perfect square. So k must be a perfect square.
      The existence of a case where a+b is minimal isn't assumed as there always exist A,B satisfying that.
      Since only the existence of (A,B) is necessary here, we are fine

    • @kenthchen
      @kenthchen Před 3 lety +2

      ​@@pbj4184 Really good explanation, the key here is that choosing the minimum solution is what leads to the contradiction later. The minimum solution isn't one case that was checked, but it proves every case.

    • @pbj4184
      @pbj4184 Před 3 lety +2

      @@kenthchen Something true must work for all cases. So if it doesn't work for the case where A+B is minimal, it isn't true. And since k can only either be perfect or non-perfect square and we showed it cannot be a non-perfect square (as that leads to a contradiction), it must be a perfect square.

    • @rakeshpatil6939
      @rakeshpatil6939 Před 3 lety

      @@pbj4184 sir can you please explain me the basis of assumption @4:58
      "That since k is not a perfect square
      Surely A1 is not equal to 0

  • @dannamilenamedranoquintero5866

    Why the contradicción say that k has to be a perfect square?

  • @user-zf4si4go7t
    @user-zf4si4go7t Před měsícem

    Vieta jumping and we done

  • @Red-Brick-Dream
    @Red-Brick-Dream Před 10 měsíci +1

    I hate how these "Olympiad" problems rely so much on niche knowledge and parlour tricks that half the professors don't even know. It's like playing football on a minefield and then blaming the players for their random bad luck.

  • @GauravPandeyIISc
    @GauravPandeyIISc Před 3 lety

    Effectively, this proves that (a^2 + b^2)/(ab + 1) = b^2 if a>=b and a,b are both positive integers. Solving the above, we get that (b^3, b) are the only solutions to the above equation.

    • @Simio_Da_Tundra
      @Simio_Da_Tundra Před rokem +1

      No, it doesn't. This proves that for the case where a+b is minimal, then (a^2+b^2)/(ab+1)=b^2 for a>=b. If a+b is not minimal, then there is no contradiction in finding another solution with a smaller sum.

    • @spiderjerusalem4009
      @spiderjerusalem4009 Před 7 měsíci

      ​@@Simio_Da_Tundrahow would taking sum a+b conclude that?

  • @NitishKumar-qn7cj
    @NitishKumar-qn7cj Před 3 lety

    Dear mathematician, go through the link for legendry ans

  • @kokala2950
    @kokala2950 Před 8 měsíci

    BULGARIA MENTIONED RAHHHHH🦁🦁🦁🦁🦁

  • @glitser2021
    @glitser2021 Před 9 měsíci +1

    Can this be done via Mathematical Induction?